Diễn Đàn MathScopeDiễn Đàn MathScope
  Diễn Đàn MathScope
Ghi Danh Hỏi/Ðáp Thành Viên Social Groups Lịch Ðánh Dấu Ðã Ðọc

Go Back   Diễn Đàn MathScope > Sơ Cấp > Hình Học > Các Bài Toán Đã Được Giải

News & Announcements

Ngoài một số quy định đã được nêu trong phần Quy định của Ghi Danh , mọi người tranh thủ bỏ ra 5 phút để đọc thêm một số Quy định sau để khỏi bị treo nick ở MathScope nhé !

* Nội quy MathScope.Org

* Một số quy định chung !

* Quy định về việc viết bài trong diễn đàn MathScope

* Nếu bạn muốn gia nhập đội ngũ BQT thì vui lòng tham gia tại đây

* Những câu hỏi thường gặp

* Về việc viết bài trong Box Đại học và Sau đại học


Trả lời Gởi Ðề Tài Mới
 
Ðiều Chỉnh Xếp Bài
Old 17-12-2007, 06:58 PM   #1
duongchinh_k41
+Thành Viên+
 
duongchinh_k41's Avatar
 
Tham gia ngày: Nov 2007
Đến từ: Tình yêu toán
Bài gởi: 233
Thanks: 10
Thanked 16 Times in 14 Posts
mòi mọi người

cho tam giác ABC có (I) là tâm đường tròn nội tiếp .Gọi M là trung điểm của BC .AM cắt đường tròn (I) tại E,F .Qua E,F kẻ đường thẳng // BC cắt(I) tại J,K .Dặt $N={EK}/cap{FJ}. $CMR AN//BC
Bài này mình đã giải trên toanthpt nhung cáchgiai k0 đẹp
[RIGHT][I][B]Nguồn: MathScope.ORG[/B][/I][/RIGHT]
 
duongchinh_k41 is offline   Trả Lời Với Trích Dẫn
Old 17-12-2007, 10:25 PM   #2
PDatK40SP
+Thành Viên+
 
Tham gia ngày: Dec 2007
Bài gởi: 109
Thanks: 0
Thanked 4 Times in 4 Posts
Đường tròn $(I) $ tiếp xúc $BC,CA,AB $ tại $X,Y,Z $
$P \in YZ \cap AM $
Lấy $Q $ trên $YZ $ sao cho $AQ $ song song $BC $
Ta có $A(BCMQ)=-1 \Rightarrow (YZPQ)=-1 $
Vậy đường đối cực của $P $ đi qua $Q $. Hiển nhiên nó còn đi qua $A $, do đó $AQ $ là đường đối cực của $P $
Suy ra $IP \perp AQ $ , từ đó $P \in EF \cap KJ $
Vì $N \in EK \cap FJ, P \in EF \cap KJ $ nên $N $ thuộc đường đối cực của $P $, nghĩa là $N \in AQ $, hay $AN \parallel BC $
[RIGHT][I][B]Nguồn: MathScope.ORG[/B][/I][/RIGHT]
 
PDatK40SP is offline   Trả Lời Với Trích Dẫn
Old 18-12-2007, 05:24 PM   #3
duongchinh_k41
+Thành Viên+
 
duongchinh_k41's Avatar
 
Tham gia ngày: Nov 2007
Đến từ: Tình yêu toán
Bài gởi: 233
Thanks: 10
Thanked 16 Times in 14 Posts
Anh Pdat giải hay quá
Cảm ơn anh
[RIGHT][I][B]Nguồn: MathScope.ORG[/B][/I][/RIGHT]
 
duongchinh_k41 is offline   Trả Lời Với Trích Dẫn
Trả lời Gởi Ðề Tài Mới

Bookmarks

Ðiều Chỉnh
Xếp Bài

Quuyền Hạn Của Bạn
You may not post new threads
You may not post replies
You may not post attachments
You may not edit your posts

BB code is Mở
Smilies đang Mở
[IMG] đang Mở
HTML đang Tắt

Chuyển đến


Múi giờ GMT. Hiện tại là 05:04 PM.


Powered by: vBulletin Copyright ©2000-2024, Jelsoft Enterprises Ltd.
Inactive Reminders By mathscope.org
[page compression: 42.99 k/47.46 k (9.41%)]